Difference between revisions of "2007 AMC 8 Problems/Problem 15"

(Solution)
(Solution)
Line 10: Line 10:
 
According to the given rules, every number needs to be positive. Since <math>c</math> is always greater than <math>b</math>, adding a positive number (<math>a</math>) to <math>c</math> will always make it greater than <math>b</math>.
 
According to the given rules, every number needs to be positive. Since <math>c</math> is always greater than <math>b</math>, adding a positive number (<math>a</math>) to <math>c</math> will always make it greater than <math>b</math>.
  
Therefore, the answer is <math>\boxed{\textbf{(c)}\ a+b<c}</math>
+
Therefore, the answer is <math>\boxed{\textbf{(a)}\ a+c<b}</math>
  
 
==See Also==
 
==See Also==
 
{{AMC8 box|year=2007|num-b=14|num-a=16}}
 
{{AMC8 box|year=2007|num-b=14|num-a=16}}
 
{{MAA Notice}}
 
{{MAA Notice}}

Revision as of 11:51, 24 August 2019

Problem

Let $a, b$ and $c$ be numbers with $0 < a < b < c$. Which of the following is impossible?

$\mathrm{(A)} \ a + c < b  \qquad \mathrm{(B)} \ a \cdot b < c \qquad \mathrm{(C)} \ a + b < c \qquad \mathrm{(D)} \ a \cdot c < b \qquad \mathrm{(E)}\frac{b}{c} = a$

Solution

According to the given rules, every number needs to be positive. Since $c$ is always greater than $b$, adding a positive number ($a$) to $c$ will always make it greater than $b$.

Therefore, the answer is $\boxed{\textbf{(a)}\ a+c<b}$

See Also

2007 AMC 8 (ProblemsAnswer KeyResources)
Preceded by
Problem 14
Followed by
Problem 16
1 2 3 4 5 6 7 8 9 10 11 12 13 14 15 16 17 18 19 20 21 22 23 24 25
All AJHSME/AMC 8 Problems and Solutions

The problems on this page are copyrighted by the Mathematical Association of America's American Mathematics Competitions. AMC logo.png